Alternating series test for convergence

Click For Summary
The discussion revolves around understanding the alternating series test for convergence, specifically the derivation of terms like 1/2 and n/(n + 1). Participants express confusion about whether it suffices to show that an + 1 ≤ an for convergence. It is noted that starting with the result and working backward can clarify the logic. The importance of comparing the magnitudes of sequence elements is emphasized, as the powers of -1 do not affect this comparison. The conversation concludes with an acknowledgment of the need to check the sign separately, which is considered straightforward in this context.
Maddie1609
Messages
81
Reaction score
11

Homework Statement



20151026_230927.jpg


Homework Equations



20151026_230940.jpg


The Attempt at a Solution



I don't get how they got what's stated in the above picture. Where does 1/2 and n/(n + 1) come from? Can't you just show that an + 1 ≤ an?
 
Physics news on Phys.org
Maddie1609 said:
Where does 1/2 and n/(n + 1) come from? Can't you just show that an + 1 ≤ an?
That's what they do. The logic is easier to follow if you start with the result and reduce this to something true (look at the steps in reverse order), but this direction works as well. The first line is then a clever guess what will be needed later (and the statement is clearly true).
 
mfb said:
That's what they do. The logic is easier to follow if you start with the result and reduce this to something true (look at the steps in reverse order), but this direction works as well. The first line is then a clever guess what will be needed later (and the statement is clearly true).
A lot easier to follow that direction, thanks! The final step has 2n and 2n - 1 instead of (-2)n and (-2)n - 1 which I don't get.
 
The test compares the magnitude of the sequence elements only, powers of -1 don't change the magnitude.
The sign has to be checked separately (easy here).
 
  • Like
Likes Maddie1609
mfb said:
The test compares the magnitude of the sequence elements only, powers of -1 don't change the magnitude.
The sign has to be checked separately (easy here).
Oh okay! Thank you :smile:
 
Question: A clock's minute hand has length 4 and its hour hand has length 3. What is the distance between the tips at the moment when it is increasing most rapidly?(Putnam Exam Question) Answer: Making assumption that both the hands moves at constant angular velocities, the answer is ## \sqrt{7} .## But don't you think this assumption is somewhat doubtful and wrong?

Similar threads

  • · Replies 1 ·
Replies
1
Views
2K
  • · Replies 7 ·
Replies
7
Views
2K
  • · Replies 3 ·
Replies
3
Views
1K
  • · Replies 2 ·
Replies
2
Views
2K
  • · Replies 5 ·
Replies
5
Views
2K
Replies
14
Views
2K
  • · Replies 6 ·
Replies
6
Views
2K
  • · Replies 22 ·
Replies
22
Views
3K
  • · Replies 5 ·
Replies
5
Views
2K
  • · Replies 7 ·
Replies
7
Views
2K